-
Notifications
You must be signed in to change notification settings - Fork 0
/
hw2.tex
341 lines (302 loc) · 17 KB
/
hw2.tex
1
2
3
4
5
6
7
8
9
10
11
12
13
14
15
16
17
18
19
20
21
22
23
24
25
26
27
28
29
30
31
32
33
34
35
36
37
38
39
40
41
42
43
44
45
46
47
48
49
50
51
52
53
54
55
56
57
58
59
60
61
62
63
64
65
66
67
68
69
70
71
72
73
74
75
76
77
78
79
80
81
82
83
84
85
86
87
88
89
90
91
92
93
94
95
96
97
98
99
100
101
102
103
104
105
106
107
108
109
110
111
112
113
114
115
116
117
118
119
120
121
122
123
124
125
126
127
128
129
130
131
132
133
134
135
136
137
138
139
140
141
142
143
144
145
146
147
148
149
150
151
152
153
154
155
156
157
158
159
160
161
162
163
164
165
166
167
168
169
170
171
172
173
174
175
176
177
178
179
180
181
182
183
184
185
186
187
188
189
190
191
192
193
194
195
196
197
198
199
200
201
202
203
204
205
206
207
208
209
210
211
212
213
214
215
216
217
218
219
220
221
222
223
224
225
226
227
228
229
230
231
232
233
234
235
236
237
238
239
240
241
242
243
244
245
246
247
248
249
250
251
252
253
254
255
256
257
258
259
260
261
262
263
264
265
266
267
268
269
270
271
272
273
274
275
276
277
278
279
280
281
282
283
284
285
286
287
288
289
290
291
292
293
294
295
296
297
298
299
300
301
302
303
304
305
306
307
308
309
310
311
312
313
314
315
316
317
318
319
320
321
322
323
324
325
326
327
328
329
330
331
332
333
334
335
336
337
338
339
340
341
\documentclass[boxes,pages,color=SeaGreen]{homework}
\hypersetup{
colorlinks=true,
urlcolor=SeaGreen!60!black,
linkcolor=Bittersweet
}
\newcommand{\collab}[1]{\footnote{\href{mailto:#1}{\texttt{#1}}}}
\name{Nate Stemen}
\studentid{20906566}
\email{[email protected]}
\term{Fall 2021}
\course{Theory of Quantum Information}
\courseid{QIC 820}
\hwnum{2}
\duedate{Nov 5, 2021}
\hwname{Assignment}
\usepackage{physics}
\usepackage{relsize}
\usepackage{macros}
\usepackage{cleveref}
\usepackage{multirow}
\usepackage{booktabs}
\usepackage{todonotes}
%-----------------------------------------------------------------------------%
% Macros
%-----------------------------------------------------------------------------%
\renewcommand{\vec}{\operatorname{vec}}
\newcommand{\X}{\mathcal{X}}
\newcommand{\Y}{\mathcal{Y}}
\newcommand{\Z}{\mathcal{Z}}
\newcommand{\W}{\mathcal{W}}
\newcommand{\V}{\mathcal{V}}
\newcommand{\U}{\mathcal{U}}
\renewcommand{\P}{\mathcal{P}}
\newcommand{\Fid}{\operatorname{F}}
\newcommand{\Lin}{\mathrm{L}}
\newcommand{\Trans}{\mathrm{T}}
\newcommand{\Pos}{\mathrm{Pos}}
\newcommand{\Herm}{\mathrm{Herm}}
\newcommand{\Channel}{\mathrm{C}}
\newcommand{\Unitary}{\mathrm{U}}
\newcommand{\Density}{\mathrm{D}}
\newcommand{\CP}{\mathrm{CP}}
\newcommand{\triplenorm}[1]{
\lvert\!\lvert\!\lvert #1
\rvert\!\rvert\!\rvert}
\begin{document}
\def\arraystretch{1.2}
%-----------------------------------------------------------------------------%
\begin{problem}
Suppose that $\X$ and $\Y$ are complex Euclidean spaces and $M\in\Lin(\Y,\X)$
is a given operator.
Define a map $\Phi\in\Trans(\X\oplus\Y)$ as
\[
\Phi\mqty(
X & Z \\
W & Y
) = \mqty(
X & 0 \\
0 & Y
)
\]
for all $X\in\Lin(\X)$, $Y\in\Lin(\Y)$, $Z\in\Lin(\Y,\X)$, and
$W\in\Lin(\X,\Y)$ (i.e., $\Phi$ zeroes out the off-diagonal blocks of a
$2\times 2$ block operator of the form suggested in the equation), and
consider the semidefinite program
\[
\qty(
\Phi,
\frac{1}{2}\mqty(0 & M \\ M^* & 0),
\mqty(\1_{\X} & 0 \\ 0 & \1_{\Y})
).
\]
\begin{parts}
\part
Express the primal and dual problems associated with this semidefinite
program in simple, human-readable terms.
(There is no single, well-defined answer to this part of the problem---just
do your best to make the primal and dual problems look as simple and
elegant as possible.)\label{part:1a}
\part
Prove that strong duality holds for this semidefinite program.\label{part:1b}
\part
What is the optimal value of this semidefinite program?\label{part:1c}
\end{parts}
\end{problem}
\noindent Solution completed in collaboration with Alev Orfi,\collab{[email protected]} and Muhammad Usman Farooq.\collab{[email protected]}
{\noindent\color{SeaGreen!30}\rule{\textwidth}{1.5pt}}
\begin{solution}
\ref{part:1a}
Given the nice form of $\Phi$ we can start by doing some algebra to simplify the program.
Starting with the definition of a semidefinite program we have
\begin{center}
\begin{tabular}{rl}
$\underset{X}{\text{maximize}}$: & $\left\langle \frac{1}{2}\spmqty{0 & M \\ M^* & 0}, X\right\rangle$ \\ \cmidrule(lr{1em}){2-2}
\multirow{2}{*}{subject to:} & $\Phi(X) = \spmqty{\1_\X & 0 \\ 0 & \1_\Y}$ \\
& $X\in\Pos(\X\oplus\Y)$.
\end{tabular}
\end{center}
Taking $X = \smqty(A & B \\ C & D)$ together with the condition that $\Phi(X) = \smqty(A & 0 \\ 0 & D) = \smqty(\1_\X & 0 \\ 0 & \1_\Y)$ we see that $A = \1_\X$ and $B = \1_\Y$.
Since $X\in\Pos(\X)$ it must also be hermitian ($X = X^*$), and hence $C = B^*$ making $X = \smqty(\1_\X & B \\ B^* & \1_\Y)$.
We can now expand out the inner product we are trying to maximize as follows (temporarily omitting the $\frac{1}{2}$):
\begin{align*}
\left\langle\spmqty{0 & M \\ M^* & 0}, \spmqty{\1_\X & B \\ B^* & \1_\Y}\right\rangle & = \tr\qty[\spmqty{MB^* & M \\ M^* & M^*B}] \\
& = \tr(MB^*) + \tr(M^*B) \\
& = \langle B, M\rangle + \langle B^*, M^*\rangle
\end{align*}
This allows us to rewrite the primal problem as
\begin{center}
\begin{tabular}{rl}
$\underset{B}{\text{maximize}}$: & $\frac{1}{2}\langle B, M\rangle + \frac{1}{2}\langle B^*, M^*\rangle$ \\ \cmidrule(lr{1em}){2-2}
\multirow{2}{*}{subject to:} & $\spmqty{\1_\X & B \\ B^* & \1_Y} \in \Pos(\X\oplus\Y)$ \\
& $B\in\Lin(\Y, \X)$.
\end{tabular}
\end{center}
In order to understand the dual problem we have to first know what $\Phi^*$ is.
This can be calculated by the definition of a adjoint map: $\langle\Phi(X), Y\rangle = \langle X, \Phi^*(Y)\rangle$.
\begin{align*}
\langle\Phi\spmqty{X & Z \\ W & Y}, \spmqty{A & B \\ C & D}\rangle & = \tr(\spmqty{X^*A & \cdot \\ \cdot & Y^*D}) = \tr(X^*A) + \tr(Y^*D) = \langle X, A\rangle + \langle Y , D \rangle \\
\langle\spmqty{X & Z \\ W & Y}, \Phi^*\spmqty{A & B \\ C & D}\rangle & = \tr(\spmqty{X^*A' + W^*C' & \cdot \\ \cdot & Z^*B' + Y^*D'})
\end{align*}
In order for this to be equal to $\tr(X^*A) + \tr(Y^*D)$ we must have $A' = A$, $D' = D$, as well as $C' = 0 = B'$.
This gives us the action of $\Phi^*$ as $\Phi^*\smqty(A & B \\ C & D) = \smqty(A & 0 \\ 0 & D)$ and hence $\Phi = \Phi^*$.
To simplify the dual problem we first write it in it's full generality:
\begin{center}
\begin{tabular}{rl}
$\underset{Y}{\text{minimize}}$: & $\left\langle \spmqty{\1_\X & 0 \\ 0 & \1_\Y}, Y\right\rangle$ \\ \cmidrule(lr{1em}){2-2}
\multirow{2}{*}{subject to:} & $\Phi(Y) \geq \frac{1}{2}\spmqty{0 & M \\ M^* & 0}$ \\
& $Y\in\Herm(\X\oplus \Y)$.
\end{tabular}
\end{center}
With a little bit of algebra, together with the fact that $Y\in\Herm(\X\oplus\Y)$ this problem transforms to
\begin{center}
\begin{tabular}{rl}
$\underset{A, D}{\text{minimize}}$: & $\tr(A) + \tr(D)$ \\ \cmidrule(lr{1em}){2-2}
\multirow{2}{*}{subject to:} & $\spmqty{A & -\frac{1}{2}M \\ -\frac{1}{2}M^* & D}\in\Pos(\X\oplus \Y)$ \\
& $A\in\Herm(\X)$ \\
& $D\in\Herm(\Y)$.
\end{tabular}
\end{center}
First we redefine $\tilde{A} = \frac{1}{2}A$ and $\tilde{D} = \frac{1}{2}D$.
We can also show $\tilde{A}\in\Pos(\X)$ as follows.
Since $T = \smqty(\tilde{A} & - M \\ -M^* & \tilde{D})\in\Pos(\X\oplus\Y)$ we know $x^*Tx\geq 0$ for all $x\in\X\oplus\Y$.
In particular this is true for $x = \smqty(x_0 \\ 0)$ and $x = \smqty(0 \\ y_0)$ separately.
This shows $x_0^*\tilde{A}x_0 \geq 0$ for all $x_0\in\X$ and $y_0^*\tilde{D}y_0 \geq 0$ for all $y_0\in\Y$.
Thus our final dual problem has simplified to
\begin{center}
\begin{tabular}{rl}
$\underset{A, D}{\text{minimize}}$: & $\frac{1}{2}\tr(A) + \frac{1}{2}\tr(D)$ \\ \cmidrule(lr{1em}){2-2}
\multirow{2}{*}{subject to:} & $\spmqty{A & -M \\ -M^* & D}\in\Pos(\X\oplus \Y)$ \\
& $A\in\Pos(\X)$ \\
& $D\in\Pos(\Y)$.
\end{tabular}
\end{center}
\ref{part:1b}
We can now apply Lemma 3.18 to our primal problem.
This means $\smqty(\1_\X & B \\ B^* & \1_\Y)\in\Pos(\X\oplus\Y)$ if and only if $B = \sqrt{\1_\X}K\sqrt{\1_\Y} = K$ for some $K\in\Lin(\Y,\X)$ with $\norm{K}\leq 1$.
Thus $B$ is bounded $\norm{B} \leq 1$ and since $M$ is fixed $\alpha$ (optimum value for primal problem) must be finite.
In order to use Slater's theorem for semidefinite programs I need to show there exists a $Y\in\Herm(\X\oplus\Y)$ such that $\Phi^*(Y) \geq \frac{1}{2}\smqty(0 & M \\ M^* & 0)$.
That is we need to show for $A\in\Herm(\X)$ and $D\in\Herm(\Y)$ that
\begin{equation*}
\Phi\smqty(A & B \\ C & D) - \frac{1}{2}\smqty(0 & M \\ M^* & 0) \approx \smqty(A & - M \\ -M^* & D) \in \Pos(\X\oplus\Y).
\end{equation*}
I've used $\approx$ to get rid of the halves.
This if the exact condition we have in the dual problem, but I'm stuck as to where to go from here.
\ref{part:1c}
The optimal value for this semidefinite program is the spectral norm: $\norm{M}_1$.
\end{solution}
%-----------------------------------------------------------------------------%
\begin{problem}
Let $\X$ be a complex Euclidean space, and define
\[
\delta(P,Q) = \sqrt{\tr(P) + \tr(Q) - 2 \Fid(P,Q)}
\]
for all positive semidefinite operators $P,Q\in\Pos(\X)$.
Prove that $\delta$ satisfies these three properties:
\begin{parts}
\part
$\delta(P,Q) \geq 0$ for all $P,Q\in\Pos(\X)$, with $\delta(P,Q) = 0$ if
and only if $P = Q$.\label{part:2a}
\part
$\delta(P,Q) = \delta(Q,P)$ for all $P,Q\in\Pos(\X)$.\label{part:2b}
\part
$\delta(P,Q) \leq \delta(P,R) + \delta(R,Q)$
for all $P,Q,R\in\Pos(\X)$.\label{part:2c}
\end{parts}
(These are the three defining properties of a \emph{metric}.)
Hint: to prove that property (c) holds, first prove that if
$\Y$ is a complex Euclidean space with $\dim(\Y)\geq\dim(\X)$,
and $u\in\X\otimes\Y$ is any vector satisfying $\tr_{\Y}(u u^{\ast}) = P$,
then
\[
\delta(P,Q) = \min_{v\in\X\otimes\Y}\qty\big{\norm{u - v} : \tr_{\Y}(v v^*) = Q}.
\]
\end{problem}
\begin{solution}
\ref{part:2a}
By the symmetry of $\delta$ (proved next) we can without loss of generality take $\tr(P)\geq \tr(Q)$.
Then, because $\tr(X) \geq 0$ for all $X\in\Pos(\X)$ we have
\begin{align*}
\qty(\sqrt{\tr(P)} - \sqrt{\tr(Q)})^2 & \geq 0 \\
\tr(P) + \tr(Q) - 2\sqrt{\tr(P)\tr(Q)} & \geq 0 \\
\tr(P) + \tr(Q) & \geq 2\sqrt{\tr(P)\tr(Q)} \\
& \geq 2\Fid(P, Q) \tag{Proposition 3.12:6}
\end{align*}
By the last inequality we have that the radicand is greater than or equal to 0, and hence $\delta(P, Q) \geq 0$.
In order for $\delta(P, Q) = 0$, we must have both $\tr(P) = \tr(Q)$ and $Q = \lambda P$ again by proposition 3.12:6.
Then we have $\tr(P) = \tr(\lambda P) = \lambda \tr(P)$ and hence $\lambda = 1$ implying $P = Q$.
\ref{part:2b}
The symmetry of $\delta$ follows immediately from Proposition 3.12:2 which states that $\Fid(P, Q) = \Fid(Q, P)$.
\begin{equation*}
\delta(P, Q) = \sqrt{\tr(P) + \tr(Q) - 2\Fid(P, Q)} = \sqrt{\tr(Q) + \tr(P) - 2\Fid(Q, P)} = \delta(Q, P)
\end{equation*}
\ref{part:2c}
We start by proving the hint.
Let $u\in\X\otimes\Y$ be a purification of $P$.
\begin{align*}
\delta(P, Q)^2 & = \min_v\qty{\langle u - v, u - v\rangle: \tr_\Y(vv^*) = Q} \\
& = \min_v \qty{\norm{u}^2 + \norm{v}^2 - 2\Re(\langle u, v\rangle)} \\
& = \tr(P) + \tr(Q) - 2\max_v\qty{\Re(\langle u, v\rangle)} \\
& \leq \tr(P) + \tr(Q) - 2\max_v\qty{\abs{\langle u, v\rangle}} \\
& = \tr(P) + \tr(Q) - 2\Fid(P, Q) \tag{\small Uhlmann's theorem}
\end{align*}
Thus in order for the hint to be true (which it has to, that would be illegal otherwise) we must have $\max_v\qty{\Re(\langle u, v\rangle)} = \max_v\qty{\abs{\langle u, v \rangle}}$ which I cannot see why must be true.
With the hint proved, let $p, q, r\in\X\otimes\Y$ be a purifications of $P$, $Q$ and $R$ respectively.
\begin{align*}
\delta(P, Q) & = \min_{q}\qty{\norm{p - q}} \\
& = \min_{q}\qty{\norm{p - q - r + r}} \\
& \leq \min_{q}\qty{\norm{p - r} + \norm{r - q}} \\
& \leq \min_{r}\qty{\norm{p - r}} + \min_{q}\qty{\norm{r - q}} \\
& = \delta(P, R) + \delta(R, Q)
\end{align*}
\end{solution}
%-----------------------------------------------------------------------------%
\begin{problem}
Let $\Phi\in\Trans(\X,\Y)$ be a map, for complex Euclidean spaces $\X$ and
$\Y$.
Prove that
\[
\triplenorm{\Phi}_1 =
\max_{\rho_0,\rho_1\in\Density(\X)}
\norm{
\qty\big( \1_{\Y} \otimes \sqrt{\rho_0} ) J(\Phi)
\qty\big( \1_{\Y} \otimes \sqrt{\rho_1} )
}_1.
\]
\end{problem}
\noindent Solution completed in collaboration with Mohammad Ayyash,\collab{[email protected]} and Nicholas Zutt.\collab{[email protected]}
{\noindent\color{SeaGreen!30}\rule{\textwidth}{1.5pt}}
\begin{solution}
We begin with a lemma.
\begin{lemma}
Let $\X = \C^\Sigma$ and $\Y = \C^\Pi$ be complex euclidean spaces.
For all $A, B\in\Lin(\X)$ and $\Phi\in\Trans(\X, \Y)$ we have
\begin{equation*}
\qty(\1_\Y\otimes A^\intercal)J(\Phi)\qty(\1_\Y\otimes\overline{B}) = \qty(\Phi\otimes\1_{\Lin(\X)})(\vec(A)\vec(B)^*).
\end{equation*}
\end{lemma}
\begin{proof}
We start by expanding the Choi matrix using the definition: $J(\Phi) = \sum_{a,b\in\Sigma}\Phi(E_{a,b})\otimes E_{a,b}$.
\begin{equation*}
\qty(\1_\Y\otimes A^\intercal)J(\Phi)\qty(\1_\Y\otimes\overline{B}) = \sum_{a,b\in\Sigma}\Phi(E_{a,b})\otimes A^\intercal E_{a,b}\overline{B}
\end{equation*}
Let's calculate $A^\intercal E_{a,b}\overline{B}$ expanding $A$ and $B$ in the standard basis as $A = \sum_{a,b\in\Sigma}\alpha_{a,b}E_{a,b}$ and $B = \sum_{a,b\in\Sigma}\beta_{a,b}E_{a,b}$.
\begin{align*}
A^\intercal E_{a,b}\overline{B} & = \qty[\sum_{c,d\in\Sigma}\alpha_{d,c}E_{c,d}]E_{a,b}\qty[\sum_{e,f\in\Sigma}\overline{\beta_{e,f}}E_{e,f}] \\
& = \sum_{c,d,e,f\in\Sigma}\alpha_{d,c}\overline{\beta_{e,f}}\,E_{c,d}\,E_{a,b}\,E_{e,f} \\
& = \sum_{c,f\in\Sigma}\alpha_{a,c}\overline{\beta_{b,f}}E_{c,f}
\end{align*}
Thus together we have
\begin{align*}
\qty(\1_\Y\otimes A^\intercal)J(\Phi)\qty(\1_\Y\otimes\overline{B}) & = \sum_{a,b,c,f\in\Sigma}\alpha_{a,c}\,\overline{\beta_{b,f}}\;\Phi(E_{a,b})\otimes E_{c,f} \\
& = \qty(\Phi\otimes\1_{\Lin(\X)})\qty[\sum_{a,b,c,f\in\Sigma}\alpha_{a,c}\,\overline{\beta_{b,f}}\;E_{a,b}\otimes E_{c,f}]
\end{align*}
We we must show the argument to $\Phi\otimes\1_{\Lin(\X)}$ is equal to $\vec(A)\vec(B)^*$.
\begin{align*}
\sum_{a,b,c,f\in\Sigma}\alpha_{a,c}\,\overline{\beta_{b,f}}\;E_{a,b}\otimes E_{c,f} & = \qty[\sum_{a,c\in\Sigma}\alpha_{a,c}\;e_a\otimes e_c]\qty[\sum_{b,f\in\Sigma}\overline{\beta_{b,f}}\;e_b^*\otimes e_f^*] \\
& = \qty[\sum_{a,c\in\Sigma}\alpha_{a,c}\;e_a\otimes e_c]\qty[\sum_{b,f\in\Sigma}\beta_{b,f}\;e_b\otimes e_f]^* \\
& = \vec(A)\vec(B)^*
\end{align*}
As desired.
\end{proof}
By Proposition 3.44:1 we know the completely bounded trace norm can be written $\triplenorm{\Phi}_1 = \max\qty{\norm{(\Phi\otimes\1_{\Lin(\X)})(uv^*)}_1: u,v\in\mathcal{S}(\X\otimes\X)}$.
Applying the above lemma to this statement transforms it to
\begin{equation*}
\triplenorm{\Phi}_1 = \max_{A,B\in\Lin(\X)}\qty{\norm{\qty(\1_\Y\otimes A^\intercal)J(\Phi)\qty(\1_\Y\otimes\overline{B})}_1 : \norm{A}_2 = 1 = \norm{B}_2}
\end{equation*}
because $\vec(A)\in\mathcal{S}(\X\otimes\X)$ implies $\sqrt{\sum_{a,b\in\Sigma}\alpha_{a,b}^2} = 1$ which is exactly the expression for $\norm{A}_2$.
Applying the polar decomposition to $A$ (and $B$) we have $A = UP$ where $U\in\Unitary(\X)$ and $P\in\Pos(\X)$.
Using the fact that $\norm{X}_2 = \sqrt{\tr(X^*X)}$ we have
\begin{equation*}
1 = \norm{A}_2 = \tr(A^*A) = \tr(P^*U^*UP) = \tr(P^*P).
\end{equation*}
Since the unitary component does not affect the norm $\norm{\cdot}_2$, we are ranging over positive semi-definite operators $\rho \defeq P^*P$ such that $\tr \rho = 1$.
That is, the maximum is attained when ranging over density operators.
We can then express $A$ in terms of this density operator as $A = \sqrt{\rho}$.
Using all these facts, together with the fact that $\sqrt{\rho}^\intercal$ can always be written as $\sqrt{\rho'}$ we have the desired equality:
\begin{equation*}
\triplenorm{\Phi}_1 = \max_{\rho_0,\rho_1\in\Density(\X)} \norm\Big{\qty(\1_\Y\otimes\sqrt{\rho_0})J(\Phi)\qty(\1_\Y\otimes\sqrt{\rho_1})}.
\end{equation*}
\end{solution}
\end{document}